Answer:
Given the function:
f(x) = the quantity of x squared plus 3x minus 4 , all over x plus 4
i,e
[tex]f(x) = \frac{x^2+3x-4}{x+4}[/tex]
or
[tex]f(x) = \frac{x^2+4x-x-4}{x+4}[/tex]
or
[tex]f(x) = \frac{x(x+4)-1(x+4)}{x+4}[/tex]
or
[tex]f(x) = \frac{(x+4)(x-1)}{x+4}[/tex]
⇒y= f(x) = x -1 ......[1]
Find the x-intercepts and y-intercepts for the equation [1] ;
y-intercepts defined as the graph crosses the y-axis;
substitute x =0 to solve for y;
y = 0-1 = -1
y = -1
x-intercepts defined as the graph crosses the x-axis;
substitute y =0 to solve for x;
0 = x-1
or
x = 1
Plot these points (0 , -1) and (1, 0) on the graph for the given function as shown below.
The slope of the line below is-1/7Write a point-slope equation of the line using the coordinates of the labeled point.
Answer:
While you do not have the coordinates included, you can use the point-slope form of the equation and input the slope. This would be:
y - y1 = -1/7(x - x1)
In this equation, simply put the ordered pair in at x1 and y1. This will give you the full point-slope equation.
Find the slope-intercept form of the equation 3x + 7y = 14.
A. 7y = 3x + 14
B. 7y = -3x + 14
C. y= (-3)/7 x+2
D. y= 3/7 x+2
The slope-intercept form:
[tex]y=mx+b[/tex]
We have:
[tex]3x+7y=14[/tex] subtract 3x from both sides
[tex]7y=-3x+14[/tex] divide both sides by 7
[tex]\boxed{y=-\dfrac{3}{7}x+2\to\boxed{C.}}[/tex]
Find the next two terms of the sequence: 80, ?40, 20, ?10, ______, ______. A) 20, 40 B) 1 2 , 1 4 C) 5, ? 5 2 D) ?30, ?40
Answer:
5, 5/2
Step-by-step explanation:
80, 40, 20, 10, ______,
80/40 = 2
40/20 = 2
We are dividing by 2 each time
10/2 =5 so the next term is 5
5/2 = 5/2
find the sum of the geometric series. 14-7+(7/2)-(7/4)+
Someone please help with these questions
Question 4:
Statement Reason
1. [tex]\overline{AC}[/tex] and [tex]\overline{BD}[/tex] bisect at O 1. Given
2. O is the midpoint of [tex]\overline{AC}[/tex] and [tex]\overline{BD}[/tex] 2. Definition of bisector
3. [tex]\overline{AO}[/tex] ≅ [tex]\overline{CO}[/tex] and [tex]\overline{BO}[/tex] ≅ [tex]\overline{DO}[/tex] 3. Definition of midpoint
4. ∠AOB ≅ ∠COD 4. Vertical Angle theorem
5. ΔAOB ≅ ΔCOD 5. SAS postulate
6. ∠A ≅ ∠C 6. CPCTC
*****************************************************************************
Question 5:
Statement Reason
1. [tex]\overline{BD}[/tex] ≅ [tex]\overline{BC}[/tex] and [tex]\overline{AD}[/tex] ≅ [tex]\overline{CD}[/tex] 1. Given
2. [tex]\overline{BD}[/tex] ≅ [tex]\overline{BD}[/tex] 2. Reflexive Property
3. ΔABD ≅ ΔCBD 3. SSS Postulate
4. ∠1 ≅ ∠2 and ∠3 ≅ ∠4 4. CPCTC
5. [tex]\overline{BD}[/tex] bisects ∠ABC and ∠ADC 5. Definition of angle bisector
HELPP! Calculate S22 for the arithmetic sequence in which a12=2.4 and the common difference is d=3.4
Answer:
S22 for the arithmetic sequence is:
First option: 15.4
Step-by-step explanation:
a12=2.4
d=3.4
S22=?
Sn=(a1+an)n/2
n=22
S22=(a1+a22)22/2
S22=(a1+a22)11
ak=aj+(k-j)d
a12=a1+(12-1)d
2.4=a1+11(3.4)
2.4=a1+37.4
Solving for a1: Subtracting 37.4 both sides of the equation:
2.4-37.4=a1+37.4-37.4
Subtracting:
-35=a1
a1=-35
a22=a12+(22-12)d
a22=2.4+10(3.4)
a22=2.4+34
a22=36.4
S22=(a1+a22)11
S22=(-35+36.4)11
S22=(1.4)11
S22=15.4
HELP!!!!!! I believe its B but I am not sure! :(
Answer:
A
Step-by-step explanation:
When solving for x as an exponent, we need to use logarithms in order to undo the operation and rearrange the terms. We use log rules to bring down the exponent and solve. Logarithms are the inverse operations to exponents and vice versa. We have one special kind of logarithm called the natural logarithm whose base is e. We write it as ln. Since our base is e here, we will use the natural logarithm to rearrange and isolate x.
[tex]e^{4x-1} =3[/tex]
We begin by applying the natural logarithm to each side.
[tex]ln(e^{4x-1}) =ln(3)[/tex]
Log rules allow use to rearrange the exponent as multiplication in front of the log.
[tex](4x-1)ln(e) =ln(3)[/tex]
ln e as an inverse simplifies to 1.
[tex](4x-1)(1)=ln(3)[/tex]
We now apply the inverse operations for subtraction and multiplication.
[tex]4x-1+1=1+ln(3)\\4x=1+ln(3)\\\frac{4x}{4} =\frac{1+ln3}{4} \\x =\frac{1+ln3}{4}[/tex]
Option A is correct.
Ms.Rochelle wants to put her 29 students into groups of 6. How many groups of 6 can she make? If she puts any remaining students in a smaller group, how many students will be in that group?
Answer:
A. 4 groups of 6 students in each group.
B. The smaller group will have 5 students.
Step-by-step explanation:
We have been given that Ms.Rochelle wants to put her 29 students into groups of 6.
A. To find the number of groups of 6 that can be made from 29 students, we will find the greatest multiple of 6 from 29.
Multiples of 6 are: 6, 12, 18, 24, 30, 36,...
We can see that the greatest multiples of 6 in 29 in 24.
[tex]\text{Number of groups that can be made with 29 students}=\frac{24}{6}[/tex]
[tex]\text{Number of groups that can be made with 29 students}=4[/tex]
Therefore, Ms. Rochelle can form 4 groups from 29 students with 6 students in each group.
B. To find the number of students in small group, we will subtract 24 from 29.
[tex]\text{Number of students in the small group}=29-24[/tex]
[tex]\text{Number of students in the small group}=5[/tex]
Therefore, If Ms. Rochelle puts any remaining students in a smaller group, the number of students in smaller group will be 5.
If U = {natural numbers less than 20} and N = {factors of 18}, what is N
This is from the answer key to this question;
N= {4, 5, 7, 8, 10, 11, 12, 13, 14, 15, 16, 17, 19.}
In triangle LNP, R is the centroid and LO = 30. Find LR and RO. Enter the answers as numbers.
Answer:
[tex]|LR|=20[/tex]
[tex]|RO|=10[/tex]
Step-by-step explanation:
The centroid divides the median in the ratio [tex]2:1[/tex].
Thus [tex]|LR|:|RO|=2:1[/tex]
This implies that,
[tex]|LO|:|RO|=3:1[/tex]
We were given that,
[tex]|LO|=30[/tex]
We substitute this value to obtain,
[tex]30:|RO|=3:1[/tex]
or
[tex]\frac{30}{|RO|}=\frac{3}{1}[/tex]
We cross multiply to get,
[tex]3|RO|=30\times 1[/tex]
We divide through by 3, to get,
[tex]|RO|=10[/tex]
We can observe from the diagram that
[tex]|LR|+|RO|=|LO|[/tex]
[tex]|LR|+10=30[/tex]
[tex]|LR|=30-10[/tex]
[tex]|LR|=20[/tex]
There are 64 pretzels in a 16-ounce bag of chocolate covered pretezel. What is the rate of pretzels per ounce?
Ron buys pens from a store. 3 packs of blue pens cost $22.50 and 4 packs of black pens cost $34.00. How much would it cost Ron to buy 2 packs of blue pens and 3 packs of black pens?
How do the graphs of these function compare to one another?
x+2= √2 solve for x will give brainliest
Answer:
Our answer is [tex]x=-2+\sqrt{2}[/tex].
Step-by-step explanation:
To solve for x, we need to isolate the variable. We can do so by subtracting 2 to both sides.
[tex]x+2=\sqrt{2} \\x+2-2=-2+\sqrt{2} \\\\x=-2+\sqrt{2} \\[/tex]
Our answer is [tex]x=-2+\sqrt{2}[/tex].
To find the decimal, calculate with a calculator.
Solve 14.2 divided by 0.5 dhow your work and explain how you new where thr decamal point whas going to be
Final answer:
To solve the division problem 14.2 divided by 0.5, you can use the long division method. The quotient is 28.4.
Explanation:
To solve the division problem 14.2 divided by 0.5, we can perform the long division method. Here's the step-by-step process:
Place the dividend, 14.2, inside the division bar.Place the divisor, 0.5, outside the division bar.Divide the first digit of the dividend, which is 1, by the divisor. The result is 2.Write down the quotient, 2, above the division bar.Multiply the quotient, 2, by the divisor, 0.5, which gives you 1.Subtract the product, 1, from the first digit of the dividend, 1, which gives you a remainder of 0.Bring down the next digit, which is 4, from the dividend.Divide the new dividend, 4, by the divisor, 0.5. The result is 8.Write down the new quotient, 8, next to the previous quotient.Multiply the new quotient, 8, by the divisor, 0.5, which gives you 4.Subtract the product, 4, from the new dividend, 4, which gives you a remainder of 0.Since there is no remainder anymore, the division is complete. The final quotient is 28.4. Therefore, 14.2 divided by 0.5 is equal to 28.4.
Jim drank 2/5 of his water bottle and John drank 3/10 of his water bottle. How much did the both boys drink?
Kris has a box of 8 crayons. Silvia, s box has 6 times of many crayons as Kris box . How many crayons are in sylvia box
10 cookies and 3 brownies cost $11.25 6 cookies and 9 brownies cost $15.75 what is the cost of one brownie?
Answer:
One brownie costs $1.25
Step-by-step explanation:
We need to solve this with simultaneous equations. First, write the equations out in two lines:
c = cookies and b = brownies.
10c + 3b = 11.25
6c + 9b = 15.75
We need to make either the two c values the same or the two b values the same. Lets change the two b values to the same by multiplying the first equation by three:
10c + 3b = 11.25
10c * 3 = 30c
3b * 3 = 9b
11.25 * 3 = 33.75
30c + 9b = 33.75
Now we can subtract the second equation from this:
30c - 6c = 24c
9b - 9b = 0
33.75 - 15.75 = 18
24c = 18
Now divide both sides by c to find 1c:
c = 0.75
Now put this back into the first equation:
10(0.75) + 3b = 11.25
7.50 + 3b = 11.25
Move the +7.50 over to the other side making it a negative:
3b = 11.25 - 7.50
3b = 3.75
Divide both sides by 3:
b = 1.25
So one brownie costs $1.25Hat is the position of 7 in the number 876,543? A. The ten-thousands place B. The hundreds place C. The tens place D. The thousands place
Need help fast please
Answer:
13. The slope for a line that is perpendicular to the line y=4x+8 is -1/4
14. The slope for a line that is perpendicular to the line x=-6 is 0
15. The equation for a line that is perpendicular to the line 8x-4y=12 and passes through the origin is y = - (1/2) x
16. The equation for a line that is perpendicular to the line y=-(1/3)x and passes through the point (0,-10) is y=3x-10
Step-by-step explanation:
13. What is the slope for a line that is perpendicular to the line y=4x+8?
y=mx+b
Comparing with the form slope-intercept, the slope of the given line is the coefficient of x, then the slope of the given line is 4.
A line perpendicular to y=4x+8 must have a slope opposite and inverse, then:
Slope of the perpendicular = - 1/4
14. What is the slope for a line that is perpendicular to the line x=-6?
The line x=-6 is a vertical line. A line perpendicular to the line x=-6 must be a horizontal line (Angle=0°), then:
Slope of the perpendicular = tan Angle = tan 0° = 0
15. Write the equation for a line that is perpendicular to the line 8x-4y=12 and passes through the origin.
8x-4y=12
Isolating y: Subtracting 8x both sides of the equation:
8x-4y-8x=12-8x
-4y=-8x+12
Dividing all the terms by -4:
-4y/(-4)= -8x/(-4)+12/(-4)
y=2x-3
The slope of the given line is 2
The slope of the perpendicular is m=-1/2 (opposite and inverse to the slope of the given line)
The perpendicular passes through the origin:
P1=(0,0)=(x1,y1)→x1=0, y1=0
Using the equation point - slope:
y-y1=m(x-x1)
Replacing the known values:
y-0=(-1/2)(x-0)
y=(-1/2)x
16. Write an equation for a line that is perpendicular to the line y=-(1/3)x and passes through the point (0,-10).
The slope of the given line is -(1/3)
The slope of the perpendicular is m=3/1→m=3 (opposite and inverse to the slope of the given line)
The perpendicular passes through the point:
P1=(0,-10)=(x1,y1)→x1=0, y1=-10
Using the equation point - slope:
y-y1=m(x-x1)
Replacing the known values:
y-(-10)=3(x-0)
y+10=3x
Isolating y: Subtracting 10 both sides of the equation:
y+10-10=3x-10
y=3x-10
Triangle ABD is congruent to Triangle DFE. If the length of line segment BC is 5 units, what is the length of line segment EF?
Answer:
EF = 5 units
Step-by-step explanation:
If ΔABC and ΔDFE are congruent, then
AB = DF
BC = FE
CA = ED
and
angles A and D are congruent
angles B and F are congruent
angles C and E are congruent
Justin's rice ball recipe uses 100 grams of rice to make 11 rice ball. Justin has 700grams of rice. How many rice balls can Justin make with 700 grams of rice?
Answer: I TINK 77 rice balls
Step-by-step explanation:
Answer:77
Step-by-step explanation:
Jenny makes and sells bracelets. She paid $12.75 for the materials to make 45 bracelets. If she sells every bracelet for $1.50 each, how much profit will she earn?
Jenny's profit per bracelet is about $1.22. If she sells all 45 bracelets, her total profit is approximately $54.77.
Explanation:The subject of this question is profit, which in this case will occur when Jenny is able to sell her bracelets for more than she paid for the materials to make them. She pays $12.75 for the materials to make 45 bracelets, meaning each bracelet costs her $12.75 / 45 = $0.283 to make. If she sells each bracelet for $1.50, that means she earns $1.50 - $0.283 = $1.217 in profit for each bracelet. If she sells all 45 bracelets, her total profit would therefore be $1.217 * 45 = $54.765. However, since we typically round money to the nearest cent, her total profit will be approximately $54.77.
Learn more about Profit Calculation here:
https://brainly.com/question/32944523
#SPJ2
If matrix A has dimensions m x n and matrix B has dimensions n x p where m, n, and p are distinct positive integers, which of the following is true?
I.the product of BA doesn't exist
II. the product of AB exists and has dimensions m x p
III. the product of AB exists and has dimensions n x n
a) I only
b) II only
c) III only
d) I and II only
e) I and III
You probably already have an idea of what a matrix is; it's a rectangular array of numbers. What they represent is a bit complicated to explain. There's a whole subject about it (see "linear algebra" for more info). Whatever they represent isn't important though, you don't need to know everything about matrices to compute their product (or whether it's even possible).
A quick definition: A matrix of dimension [tex]r[/tex]-by-[tex]c[/tex] is a matrix with [tex]r[/tex] rows and [tex]c[/tex] columns.
Matrix multiplication all comes down to an operation called the "dot product". It's defined by the sum of component-wise products of elements between two lists. What this means is, if [tex]x=\{1,2,0\}[/tex] and [tex]y=\{-1,0,3\}[/tex], then the dot product of [tex]x[/tex] and [tex]y[/tex] is
[tex]x\cdot y=(1)(-1)+(2)(0)+(0)(3)=-1+0+0=-1[/tex]
What we did was take the first elements of each list and multiplied them, and the same for the other two elements, then added them all together. Notice that the product can't be computed if [tex]x[/tex] and [tex]y[/tex] don't thave the same number of elements.
We write this product in matrix form as
[tex]\begin{bmatrix}1&2&0\end{bmatrix}\begin{bmatrix}-1\\0\\3\end{bmatrix}[/tex]
Notice the pattern here: on the left, a matrix with 1 row and 3 columns; on the right, a matrix with 3 rows and 1 column. The number of columns of the first matrix have to match the number of rows of the second.
The orientation makes a big difference. The product above returns a 1-by-1 matrix (or simply a scalar number):
[tex]\begin{bmatrix}1&2&0\end{bmatrix}\begin{bmatrix}-1\\0\\3\end{bmatrix}=\begin{bmatrix}(1)(-1)+(2)(0)+(0)(3)\end{bmatrix}=\begin{bmatrix}-1\end{bmatrix}=-1[/tex]
On the other hand, the alternate orientation would result in a 3-by-3 matrix.
[tex]\begin{bmatrix}1\\2\\0\end{bmatrix}\begin{bmatrix}-1&0&3\end{bmatrix}=\begin{bmatrix}(1)(-1)&(1)(0)&(1)(3)\\(2)(-1)&(2)(0)&(2)(3)\\(0)(-1)&(0)(0)&(0)(3)\end{bmatrix}=\begin{bmatrix}-1&0&3\\\-2&0&3\\0&0&0\end{bmatrix}[/tex]
So the number of rows of the first matrix and number of columns of the second matrix determine the number of rows and columns, respectively, of the matrix product.
I think we have enough information about matrix multiplication to answer this question. If [tex]A[/tex] has dimensions [tex]m\times n[/tex] and [tex]B[/tex] has dimensions [tex]n\times p[/tex], then the matrix product [tex]AB[/tex] exists ([tex]n[/tex] columns in [tex]A[/tex], [tex]n[/tex] rows in [tex]B[/tex]), but the matrix product [tex]BA[/tex] does not ([tex]p[/tex] columns in [tex]B[/tex], [tex]m[/tex] rows in [tex]A[/tex], but [tex]m\neq p[/tex]). So I is not true.
We know [tex]AB[/tex] exists, and with [tex]m[/tex] rows in [tex]A[/tex] and [tex] p[/tex] columns in [tex]B[/tex], we expect [tex]AB[/tex] to have [tex]m[/tex] rows and [tex]p[/tex] columns, so [tex]AB[/tex] has dimensions [tex]m\times p[/tex]. So II is true.
With dimensions [tex]r\times c[/tex], a matrix would contain [tex]rc[/tex] elements. [tex]m,n,p[/tex] are distinct, so [tex]mp\neq n^2[/tex]. So III is not true.
Answer: The correct option is
(d) I and II only.
Step-by-step explanation: Given that matrix A has dimensions m x n and matrix B has dimensions n x p where m, n, and p are distinct positive integers.
We are to select the one that is true from the following :
I. the product of BA doesn't exist
II. the product of AB exists and has dimensions m x p
III. the product of AB exists and has dimensions n x n
We know that two matrices X and Y can be multiplied if the number of columns in X is equal to the number of rows in Y.
Also, if X has dimensions a x b and Y has dimensions b x c, then the product XY is possible and it has dimensions a x c. Also, the product YX doesn't exist.
So, for the given matrices A and B, the following points are true :
(I) the product of BA doesn't exist.
(II) the product of AB exists and has dimensions m x p.
Thus, only I and II are TRUE.
Option (d) is CORRECT.
25 POINTS! EXPLAIN FOR BRAINLIEST/THANK YOU.
By the polynomial remainder theorem, a polynomial [tex]p(x)[/tex] has a factor of [tex]x-c[/tex] if [tex]p(c)=0[/tex]. So all you need to do is check the value of [tex]f(x)[/tex] at [tex]x=1,3,-3,5,-5[/tex]. You should get
[tex]f(1)=-96[/tex] (so [tex]x-1[/tex] is NOT a factor)
[tex]f(3)=-96[/tex] (so [tex]x-3[/tex] is NOT a factor)
[tex]f(-3)=f(5)=f(-5)=0[/tex] (so the last three options are factors)
Answer:
the answer is c and yeah
Step-by-step explanation:
Consider the scatter plot.
Scatter plot with line of best fit of y equals 0.75x plus five.
The line of best fit is y = 0.75x + 5.
Choose the best representation for the slope.
The slope of the line of best fit shows that each additional minute, the distance increases by 0.75 feet.
The slope of the line of best fit shows that each additional minute, the distance decreases by 5 feet.
The slope of the line of best fit shows that each additional minute, the distance decreases by 0.75 feet.
The slope of the line of best fit shows that each additional minute, the distance increases by 5 feet.
Answer:
The correct answer option is: The y-intercept of the line of best fit shows that when time started, the distance was 5 feet.
Step-by-step explanation:
We are given a scatter plot with a best fit line as shown on the given graph.
The equation of the best fit line is given by:
y = 0.75x + 5
So with the help of the equation and by looking at the given graph, we can conclude about the representation of the y intercept that the the y-intercept of the line of best fit shows that when time started, the distance was 5 feet.
Since the distance shown on the y axis is already 5 when the time started at 0 minutes.
Find X
I have no idea how to make it
Answer: 5
Step-by-step explanation:
The length of the midsegment is one-half the length of the base. So,
Base = 2(midsegment)
7x + 1 = 2(4x - 2)
7x + 1 = 8x - 4
1 = x - 4
5 = x
Write an expression to represent: Three more than the product of two and a number x.
Answer:
2x + 3
Step-by-step explanation:
product means to multiply, hence
the product of 2 and a number x = 2 × x = 2x
3 more than means add on 3, thus
expression is 2x + 3
Can someone help me with these please?
Point-slope form: y-y1 = m(x-x1)
Standard form: ax + by = c
Slope-intercept form: y = mx+b
Start by finding the slope. We know it is negative since the line is decreasing. The slope is -4/3.
To create point-slope form, we need to get one point from the graph. Let's use (3,0).
[tex]y = -\frac{4}{3}(x-3)[/tex]
To create slope-intercept form, we need the slope and the y-intercept. The y-intercept is the point where our equation crosses the y-axis. For this equation, it is 4.
[tex]y = -\frac{4}{3}x + 4[/tex]
To get standard form, solve the equation in terms of C.
Point-slope form: y = -4/3(x-3)
Slope-intercept form: y = -4/3x + 4
Standard form: 4/3x + y = 4
What is the distance between the two points (5,-2) and (-3,8)